Đến nội dung

nth1235 nội dung

Có 116 mục bởi nth1235 (Tìm giới hạn từ 05-05-2020)



Sắp theo                Sắp xếp  

#557216 Chuyên đề 4:Hình học mặt phẳng, Hình giải tích.

Đã gửi bởi nth1235 on 01-05-2015 - 08:20 trong Phương pháp tọa độ trong mặt phẳng

Trong mặt phẳng tọa độ Oxy cho tam giác ABC cân tại A, B thuộc Oy, C thuộc Ox. Đường trung tuyến qua đỉnh A là đường thẳng 2x + y -3 = 0. Biết diện tích tam giác ABC bằng 5. Tìm tọa độ đỉnh B, C.




#490336 Xin đề luyện thi Violympic quốc gia 11

Đã gửi bởi nth1235 on 03-04-2014 - 10:28 trong Tài liệu tham khảo khác

Em sắp sửa thi quốc gia violympic vào cuối tháng 4. Cho em xin tài liệu để ôn luyện ạ.




#427217 Đề thi vào lớp 10 THPT chuyên Lương Thế Vinh, Đồng Nai 2013-2014 (toán chuyên)

Đã gửi bởi nth1235 on 14-06-2013 - 17:47 trong Tài liệu - Đề thi

Anh cho em hỏi chỗ này cái, em thắc mắc thế này

 

Với 12 điểm, tạo nên một đa giác 10 đỉnh thì đỉnh thứ nhất có 12 cách chọn, đỉnh thứ 2 có 11 cách chọn,..., đỉnh thứ 10 có 3 cách chọn. Như vậy là có 12.11....3 cách chọn. Suy ra; có 12.11.....3 đa giác 10 đỉnh chứ ạ. Sao lại chỉ có 66.

Em nghĩ là em sai nên em hỏi, anh giải thích giúp em nhé

Cái này anh thấy đúng mà em vì tập hợp 12 điểm đó đã tạo thành một đa giác lồi từ trước (tức là ngta cho trước 1 đa giác lồi có 12 đỉnh) thì có 66 cách là đúng chứ. còn làm như kiểu em chắc chắn sai rồi, vì nếu nó cho trước 1 tập hợp 12 điểm thì số đa giác lồi vừa là tổ hợp, vừa là hoán vị vòng quanh đó, tổng cộng là $66.9!$




#426660 Đề thi vào lớp 10 THPT chuyên Lương Thế Vinh, Đồng Nai 2013-2014 (toán chuyên)

Đã gửi bởi nth1235 on 13-06-2013 - 09:19 trong Tài liệu - Đề thi

Đề năm nay dễ hơn rất nhiều so với đề năm ngoái,




#360349 Tìm số nguyên tố $u ; v$ thỏa mãn $u^4 + v^4$ chia hết ch...

Đã gửi bởi nth1235 on 09-10-2012 - 14:12 trong Số học

Tìm số nguyên tố $u ; v$ thỏa mãn $u^4 + v^4$ chia hết cho $(u + v)^2$



#357570 Chứng minh rằng luôn có 1 trong 2 tập đó chứa 3 số sao cho tổng của 2 trong...

Đã gửi bởi nth1235 on 29-09-2012 - 20:09 trong Tổ hợp và rời rạc

1. Chứng minh rằng : với mọi chia tập $ X= {1 ; 2 ; 3 ; 4 ; 5 ; 6 ; 7 ; 8 ; 9} $ thành hai tập con, luôn có 1 trong 2 tập đó chứa 3 số sao cho tổng của 2 trong 3 số đó bằng 2 lần số còn lại.
2. Tồn tại hay không 2 tập vô hạn A, B chứa các số nguyên không âm sao cho với số không âm $c$ bất kì nào đó, đều tìm được $a \in A , b \in B$ sao cho $c = a + b$.



#357313 Chứng minh luôn có 1 trong 2 tập đó chứa 3 số sao cho tổng của 2 trong 3 số đ...

Đã gửi bởi nth1235 on 28-09-2012 - 20:01 trong Các bài toán Đại số khác

1. Chứng minh rằng : với mọi chia tập $ X= {1 ; 2 ; 3 ; 4 ; 5 ; 6 ; 7 ; 8 ; 9} $ thành hai tập con, luôn có 1 trong 2 tập đó chứa 3 số sao cho tổng của 2 trong 3 số đó bằng 2 lần số còn lại.
2. Tồn tại hay không 2 tập vô hạn A, B chứa các số nguyên không âm sao cho với số không âm $c$ bất kì nào đó, đều tìm được $a \in A , b \in B$ sao cho $c = a + b$.



#353241 Tính: $\sqrt{2\sqrt{2\sqrt{2\sqrt...

Đã gửi bởi nth1235 on 09-09-2012 - 19:57 trong Đại số

Cái này bạn sẽ được học rõ hơn khi học sang năm lớp 11. Chính xác là phần lim. Còn bây giờ bạn có thể hiểu nôm na rằng vì số căn của nó quá nhiều nên khi bớt một dấu căn thì giá trị của nó không thay đổi.



#353229 Chứng minh rằng một phương trình bậc 3 luôn có ít nhất một nghiệm thực.

Đã gửi bởi nth1235 on 09-09-2012 - 19:15 trong Phương trình - hệ phương trình - bất phương trình

Chứng minh rằng một phương trình bậc 3 luôn có ít nhất một nghiệm thực.



#351385 [MO2013] Trận 1 - Phương trình, hệ phương trình, bất phương trình

Đã gửi bởi nth1235 on 01-09-2012 - 16:05 trong Thi giải toán Marathon dành cho học sinh Chuyên Toán 2013

Giải hệ phương trình trên tập hợp số thực :
$\begin{cases}
& \text \sqrt[8]{2.\sqrt[5]{7} - \sqrt[10]{y}} + (17 - \sqrt{37}).z^2 = 544 - 32.\sqrt{37} \\
& \text x.(9.\sqrt{1 + x^2} + 13.\sqrt{1 - x^2}) + 4\sqrt{y} = 912 \\
& \text \sqrt{(10.\sqrt{5} + 20).x.(1 - x)} + z.\sqrt[6]{8} = 10
\end{cases}$

BL :
ĐK : $0 \leq x \leq 1 ; 0 \leq y \leq 50176 (1)$
Với $0 \leq x \leq 1$, ta có :
$ x.(9.\sqrt{1 + x^2} + 13.\sqrt{1 - x^2})
= \frac{3}{2} . 3x . 2\sqrt{1 + x^2} + \frac{13}{2} . x . 2\sqrt{1 - x^2}
\leq \frac{3}{4}.[9x^2 + 4(1 + x^2)] + \frac{13}{4}.[x^2 + 4(1 - x^2)] = 16 (2)$ (Áp dụng BĐT AM - GM)
Dấu "=" xảy ra $\Leftrightarrow x = \frac{2}{\sqrt{5}} $ (thỏa $(1)$)
Mặt khác, từ $(1)$ suy ra $4.\sqrt{y} \leq 896 (3)$.
Từ $(2) , (3)$ suy ra $x.(9.\sqrt{1 + x^2} + 13.\sqrt{1 - x^2}) + 4.\sqrt{y} \leq 912$.
Mà theo đề bài, dấu bằng xảy ra nên $x = \frac{2}{\sqrt{5}} ; y = 50176$
Từ đó, thay $x = \frac{2}{\sqrt{5}} ; y = 50176$ vào hệ, suy ra $z = \sqrt{32}$
Vậy hệ có nghiệm $(x , y , z)$ duy nhất là $(\frac{2}{\sqrt{5}} ; 50176 ; \sqrt{32})$

Nhận xét : Thoạt nhìn hệ trên có vẻ khá phức tạp nhưng nếu chứng minh được bất đẳng thức $x.(9.\sqrt{1 + x^2} + 13.\sqrt{1 - x^2}) \leq 16 $ với $ 0 \leq x \leq 1$ thì bài toán trở nên đơn giản hơn.

PS : Ban tổ chức giúp em sửa Latex 1 số chỗ dấu căn hiển thị chưa rõ với. Máy nhà em ko hiểu sao sửa mãi mà không được.

OK



#351144 Topic bất đẳng thức THCS (2)

Đã gửi bởi nth1235 on 31-08-2012 - 17:19 trong Bất đẳng thức và cực trị

Viết sách đôi khi cũng phải có nhầm lẫn chứ. Bây giờ chỉ cần mình đưa ra một phản ví dụ là với k=4 thì GTLN của nó lớn hơn 2. Vậy với k=4 thì anh Cẩn sai.

Ukm, Mình xem cái cm rồi. Lâu lâu anh Cẩn cũng bất cẩn. Hì



#350836 Topic bất đẳng thức THCS (2)

Đã gửi bởi nth1235 on 30-08-2012 - 10:29 trong Bất đẳng thức và cực trị

Bài này sai ở cái đề, đọc trong link kia sẽ thấy. Còn qui nạp thì không phải lúc nào cũng đúng. Khi cho k=4 thì GTLN của nó lớn hơn hai.

Nhưng bài này mình lại thấy trong sách của anh Võ Quốc Bá Cẩn đã đưa ra bài tổng quát. Nếu sai thì tại sao có trong sách ???



#350835 Một kĩ thuật chứng minh B.Đ.T

Đã gửi bởi nth1235 on 30-08-2012 - 10:23 trong Bất đẳng thức và cực trị

2/Cho x,y dương và x+y=8
Tìm GTNN của biểu thức
P=$\dfrac{1}{x+4}+\dfrac{1}{y+4}$
3/Tìm GTNN của
E=$\dfrac{2x^{2}+12x+16}{x^{2}+6x+11}$

2. Áp dụng BĐT Cauchy - Schwarz dạng phân thức, ta có :
$P = \dfrac{1}{x+4}+\dfrac{1}{y+4} \geq \frac{(1 + 1)^2}{x + y + 8} = \frac{4}{16} = \frac{1}{4}.$
Vậy $Min P = \frac{1}{4} \Leftrightarrow x = y = 4$
3.Ta có :
$E=\dfrac{2x^{2}+12x+16}{x^{2}+6x+11} = 2 - \frac{6}{(x + 3)^2 + 2} = -1.$
Vậy $ Min P = -1 \Leftrightarrow x = -3$.



#350834 Một kĩ thuật chứng minh B.Đ.T

Đã gửi bởi nth1235 on 30-08-2012 - 10:13 trong Bất đẳng thức và cực trị

Bài 3 phần bài tập áp dụng :
Chúng ta sẽ dùng phương pháp điểm rơi để giải quyết bài toán.
Áp dụng BĐT AM - GM, ta có :
$\frac{1}{x^2 + y^2} + 4.(x^2 + y^2) \geq 4$
$\frac{3}{4xy} + 12xy \geq 6$
Cộng vế theo vế của hai BĐT trên, ta được :
$ P \geq 10 - 4.(x^2 + y^2) - 4xy = 10 - 4.(x + y)^2 - 4xy = 6 - 4xy \geq 5$ (Do $xy \geq \frac{(x + y)^2}{4} = \frac{1}{4}$)
Vậy $ Min P = 5 \Leftrightarrow x = y = \frac{1}{2}$



#350141 [MO2013] Trận 1 - Phương trình, hệ phương trình, bất phương trình

Đã gửi bởi nth1235 on 27-08-2012 - 10:20 trong Thi giải toán Marathon dành cho học sinh Chuyên Toán 2013

Chẳng bik loại bao nhiêu nữa nhưng nếu mà ko làm bài được thì loại thì có tới 31 bạn "ra đi".



#350110 [MO2013] Trận 1 - Phương trình, hệ phương trình, bất phương trình

Đã gửi bởi nth1235 on 27-08-2012 - 09:10 trong Thi giải toán Marathon dành cho học sinh Chuyên Toán 2013

Đề vừa ra đã có bạn đưa lên mathlinks mong BQT xem xét

Là ai bạn có thể nói rõ ko ???



#350109 [MO2013] Trận 1 - Phương trình, hệ phương trình, bất phương trình

Đã gửi bởi nth1235 on 27-08-2012 - 09:03 trong Thi giải toán Marathon dành cho học sinh Chuyên Toán 2013

Thống kê bao nhiêu bạn làm dc bài : (Theo thứ tự thời gian)
1) bibitsubomi 9fxshiftsolve
2) WhjteShadow
3) henry0905
4) davildark
5) hoangtrunghieu22101997
6) nthoangcute
7) triethuynhmath
8) minhtuyb
9) milinh7a
10) doxuantung97
11) chinhanh9
12) L Lawliet
13) namheo1996
14) sherlock holmes 1997
15) Trần Đức Anh @@
16) tran thanh binh dv class
17) 899225
18) luuxuan9x
19) Joker999
20) ninhxa.
Ngoài ra còn có 1 mở rộng của WhjteShadow.



#348480 Topic nhận đề PT, BPT, HPT, HBPT

Đã gửi bởi nth1235 on 20-08-2012 - 09:53 trong Thi giải toán Marathon dành cho học sinh Chuyên Toán 2013

Giải hệ phương trình trên tập hợp số thực :
$\begin{cases}
& \text \sqrt[8]{2.\sqrt[5]{7} - \sqrt[10]{y}} + (17 - \sqrt{37}).z^2 = 544 - 32.\sqrt{37} \\
& \text x.(9.\sqrt{1 + x^2} + 13.\sqrt{1 - x^2}) + 4\sqrt{y} = 912 \\
& \text \sqrt{(10.\sqrt{5} + 20).x.(1 - x)} + z.\sqrt[6]{8} = 10
\end{cases}$

BL :
ĐK : $0 \leq x \leq 1 ; 0 \leq y \leq 50176 (1)$
Với $0 \leq x \leq 1$, ta có :
$ x.(9.\sqrt{1 + x^2} + 13.\sqrt{1 - x^2})
= \frac{3}{2} . 3x . 2\sqrt{1 + x^2} + \frac{13}{2} . x . 2\sqrt{1 - x^2}
\leq \frac{3}{4}.[9x^2 + 4(1 + x^2)] + \frac{13}{4}.[x^2 + 4(1 - x^2)] = 16 (2)$ (Áp dụng BĐT AM - GM)
Dấu "=" xảy ra $\Leftrightarrow x = \frac{2}{\sqrt{5}} $ (thỏa $(1)$)
Mặt khác, từ $(1)$ suy ra $4.\sqrt{y} \leq 896 (3)$.
Từ $(2) , (3)$ suy ra $x.(9.\sqrt{1 + x^2} + 13.\sqrt{1 - x^2}) + 4.\sqrt{y} \leq 912$.
Mà theo đề bài, dấu bằng xảy ra nên $x = \frac{2}{\sqrt{5}} ; y = 50176$
Từ đó, thay $x = \frac{2}{\sqrt{5}} ; y = 50176$ vào hệ, suy ra $z = \sqrt{32}$
Vậy hệ có nghiệm $(x , y , z)$ duy nhất là $(\frac{2}{\sqrt{5}} ; 50176 ; \sqrt{32})$

Nhận xét : Thoạt nhìn hệ trên có vẻ khá phức tạp nhưng nếu chứng minh được bất đẳng thức $x.(9.\sqrt{1 + x^2} + 13.\sqrt{1 - x^2}) \leq 16 $ với $ 0 \leq x \leq 1$ thì bài toán trở nên đơn giản hơn.

PS : Ban tổ chức giúp em sửa Latex 1 số chỗ dấu căn hiển thị chưa rõ với. Máy nhà em ko hiểu sao sửa mãi mà không được.



#348475 Topic các bài toán số học dành cho các bạn chuẩn bị thi tuyển sinh 10 năm 201...

Đã gửi bởi nth1235 on 20-08-2012 - 09:30 trong Số học

Bài 21: http://diendantoanho...m-2006m12006n1/
Bài 20: đã có trên VMF

Chán thế, sao bài nào cũng có vậy, bài khác.
Bài 22 : Cho $m, n$ là các số nguyên dương thỏa mãn :
$ lcm(m , n) + gcd(m , n) = m + n$
Chứng minh rằng trong hai số $m , n$ có một số chia hết cho số còn lại.
Bài 23 : Tìm tất cả các số tự nhiên $n$ sao cho : $A = {n}^{2005} + {n}^{2006} + n^2 + n + 2$ là số nguyên tố.



#348368 Topic các bài toán số học dành cho các bạn chuẩn bị thi tuyển sinh 10 năm 201...

Đã gửi bởi nth1235 on 19-08-2012 - 20:32 trong Số học

Bài 20 : Chứng minh rằng số được thành lập bởi $3^n$ chữ số giống nhau thì chia hết cho $3^n$, trong đó $n$ là một số nguyên dương cho trước.
Bài 21 : Cho $m, n$ là hai số nguyên dương phân biệt có $(m , n) = d$ $(d$ là số nguyên dương$)$.
Tính $(2006^m + 1 , 2006^n + 1)$



#347951 Topic các bài toán số học dành cho các bạn chuẩn bị thi tuyển sinh 10 năm 201...

Đã gửi bởi nth1235 on 18-08-2012 - 18:26 trong Số học

Cho mình hỏi câu này, tập hợp $P$ là tập hợp gì mà nghe lạ quá @@
@nguyenta98:

Spoiler

Tập hợp $P$ là tập hợp số nguyên tố.
Ps : Sao toàn người quen giải bài không vậy ??? Đề nghị "Cao thủ số học" Tạ Hà Nguyên ngừng việc post lời giải lên nữa, phải để cho mấy bạn khác làm với chứ. :D



#347782 Topic các bài toán số học dành cho các bạn chuẩn bị thi tuyển sinh 10 năm 201...

Đã gửi bởi nth1235 on 18-08-2012 - 07:52 trong Số học

Post mấy bài góp vui :
$11)$ Cho $a, b, c, d$ là các số nguyên dương thỏa mãn $(a , b) = (c , d) = 1.$
Chứng minh nếu $\frac{a}{b} + \frac{c}{d}$ là số nguyên thì $b = d$.
$12)$ Tìm số có hai chữ số $\bar{ab}$ sao cho $ p = \frac{a.b}{\left| a - b \right|}$ là số nguyên tố.



#347779 Topic các bài toán số học dành cho các bạn chuẩn bị thi tuyển sinh 10 năm 201...

Đã gửi bởi nth1235 on 18-08-2012 - 07:23 trong Số học

Từ đầu bài suy ra: $A=(2005^n-1897^n)-(168^n-60^n)$
Ta có:
$2005^n-1897^n \vdots 2005-1897=108 \vdots 12$
$168^n-60^n \vdots 168-60=108 \vdots 12$
$\Rightarrow A \vdots 12$ (1)
Từ đầu bài ta cũng có: $A=(2005^n-168^n)-(1879^n-60^n)$
Ta có:
$2005^n-168^n \vdots 2005-168=1837 \vdots 167$
$1897^n-60^n \vdots 1897-60=1837 \vdots 167$
$\Rightarrow A \vdots 167$ (2)
Mà $2004=12.167$ (3)
Từ (1), (2) và (3) $\Rightarrow A \vdots 2004$
----

Spoiler

Bài này bạn nên chú ý chỗ (3). Chỗ đó chỉ đúng khi (12,167) = 1. Bạn nên lý luận thêm.
Một vd để bạn thấy nó sẽ ra sao nếu ko có điều kiện đó :
4 chia hết cho 2 và 4 chia hết cho 4 nhưng không chia hết cho 2.4 = 8.



#347399 Chứng minh $(p - 2)!$ chia hết cho $(p - 1)$

Đã gửi bởi nth1235 on 17-08-2012 - 07:50 trong Số học

Một cách làm khác mà tối qua mình nghĩ ra.
Giả sử điều ngược lại, $p$ là hợp số.
Xét hai trường hợp :
TH1 : $p = a^2$ ($a$ là số nguyên dương lớn hơn 1)
Khi đó, ta có : $ 1 < a < p $
Vì $a$ là số nguyên dương lớn hơn 1 nên $a^2 - a - 1 > 0$.
Hay $p - 1 > a$. Do đó $ (p - 1)! $ chia hết cho $a$. $(1)$
Mặt khác, theo giả thiết thì $ (p - 1)! + 1 $ chia hết cho $ p = a^2 $.
Suy ra $ (p - 1)! + 1 $ chia hết cho $a$ $(2)$
Từ $(1)$, $(2)$ suy ra 1 chia hết cho $a$, mâu thuẫn.
TH2 : $p = x.y$ ($x, y$ là các số nguyên dương khác nhau lớn hơn 1).
Không mất tính tổng quát, giả sử $ 1 < x < y < p $
Khi đó, hiển nhiên $ p - 1 \geq y > x $. Suy ra $(p - 1)!$ chia hết cho $x$. $(1)$
Mặt khác, theo giả thiết thì $ (p - 1)! + 1 $ chia hết cho $ p = x.y $.
Suy ra $ (p - 1)! + 1 $ chia hết cho $x$ $(2)$
Từ $(1)$, $(2)$ suy ra 1 chia hết cho $x$, mâu thuẫn.
Vậy ta có điều phải chứng minh.



#347311 Chứng minh $(p - 2)!$ chia hết cho $(p - 1)$

Đã gửi bởi nth1235 on 16-08-2012 - 21:16 trong Số học

Tùng ơi, cậu post lời giải cho chiều " Cho p>2. Cm nếu (p-1)! + 1 chia hết cho p thì p là snt" giùm mình với.